What is Current: Definition and 1000 Discussions

An electric current is a stream of charged particles, such as electrons or ions, moving through an electrical conductor or space. It is measured as the net rate of flow of electric charge through a surface or into a control volume. The moving particles are called charge carriers, which may be one of several types of particles, depending on the conductor. In electric circuits the charge carriers are often electrons moving through a wire. In semiconductors they can be electrons or holes. In a electrolyte the charge carriers are ions, while in plasma, an ionized gas, they are ions and electrons.The SI unit of electric current is the ampere, or amp, which is the flow of electric charge across a surface at the rate of one coulomb per second. The ampere (symbol: A) is an SI base unit Electric current is measured using a device called an ammeter.Electric currents create magnetic fields, which are used in motors, generators, inductors, and transformers. In ordinary conductors, they cause Joule heating, which creates light in incandescent light bulbs. Time-varying currents emit electromagnetic waves, which are used in telecommunications to broadcast information.

View More On Wikipedia.org
  1. J

    Response of a motor rotor to just 1/60th of a second of current

    Where can I get actual demonstration that a 1/60 sec current can move the magnet. That is. You just apply 1/60 sec current without other power in the middle of it, will the magnet move that fast in 1/60 sec? Or does motor runs because of inertia of the previous rotation?
  2. wcjy

    Magnetic field due to infinite current carrying wire in the X and Y axes

    $$B = \frac {\mu_0 I}{2 \pi r} $$ By Right-hand Grip Rule, the direction of the magnetic field by wire in y-axis is into the paper (z) while the direction of the magnetic field by wire in X-axis is upwards (+i) The answer state the Magnetic field is in the (i - y) direction though. Next...
  3. SwatiAnshu

    Calculation of Eddy Current Brake Force with a twist

    All the equations I have seen have a component that is unknown. Also, if possible do let me know if there is a software I can use to model or analyze data related to eddy currents
  4. madafo3435

    Understanding Short Circuit Current Paths in Electrical Circuits

    My reasoning is as follows: with the short circuit present, a 0 resistance can be considered between the terminals. Then, since the only objective of the short circuit current ##I_{SC}## is to give information about the Thevenin equivalent resistance and the open circuit voltage (the existence...
  5. Mr_Allod

    Magnetic Field, Field Intensity and Magnetisation

    Hello there, I've worked through this problem and I would just like to check whether I've understood it correctly. I found ##\vec H##, ##\vec B## and ##\vec M## using Ampere's Law and the above relations as I would for any thin current carrying wire and these were my answers: $$\vec H = \frac I...
  6. patric44

    Questions related to the current between the plates of a vaccum tube?

    hi guys i have an assigment of deriving the current between the plates of a vacuum tube as a function of the potential bias applied on the upper plate from what i had found this relation is called Child-Langmuir law and states that : $$I = K V_{d}^{3/2}$$ from what i find online i can derive it...
  7. S

    Checking Kirchhoff's current law

    Im trying to apply KCL to the serial c and parallel rl circuit. IZ = = = 330.562 mA but this means incorrect proof. Anyone can help please?
  8. Buzz Bloom

    I What is the best current value for Omega_r?

    The source of the 0.824×10-4 value is http://hyperphysics.phy-astr.gsu.edu/hbase/Astro/denpar.html . I also looked at the Planck 2018, paper I but could not find a value for Ωr. I searched the internet with: radiation density cosmology friedmann omega but I had no success. I also looked at...
  9. archaic

    Help! Finding Direction of Current in Magnetic Circuit

    I have nearly finished my homework, but am blocking at one of the simplest questions... I'm going to let ##\rho=0.094\,\Omega/m## Since the magnetic flux is increasing, then, by Lenz's law, the current in both loops is counter-clockwise. If we only look at the left loop, then...
  10. S

    Using Kirchhoff's Laws to Solve this Circuit with Voltage and Current Sources

    I have defined 5 currents but I can't seem to solve it. I1+1=I2 (left) I5+1=I4 (right) I2+I4=I3 upper node By KVL I have determined that I2 and I4 are equal, but I cannot determine the specific current across each resistor. Thank you for your valuable help.
  11. greg_rack

    Tension measured by a voltmeter of given bottom scale and current range

    First and foremost, I've studied voltmeters and ammeters on my own, and online there aren't many resources to do so... forgive me in advance for eventual silly questions/doubts :) I managed to calculate(intuitively) a tension of ##30.0V## per resistor, with basic procedures. However, I'm...
  12. greg_rack

    Discharge of a capacitor and current in an RC circuit

    I'm having a few troubles understanding a few things about this circuit... Firstly, what does "In light of this, the potential between the resistors is ##16V##" exactly mean? If current isn't flowing, how could there be a potential between resistors? Secondly, how does current flow with the...
  13. Mr_Allod

    Bound Bulk Current Density and Surface Current Density

    Hi there, I've worked through most of this question but I'm stuck on the final part, showing that total bulk current ##I_B## is equal and opposite to total surface current ##I_S##. I calculated ##\vec H## the normal way I would if I was looking for ##\vec B## in an infinitely long cylindrical...
  14. greg_rack

    Current in a resistor with Kirchhoff's laws

    So, I started by writing down conditions imposed by Kirchhoff's laws, coming up with a system of 3 equations with the 3 unknown currents: $$ \left\{\begin{matrix} 9V-8\Omega I_{8}+6V-4\Omega I_{4}=0 \\ 6V-4\Omega I_{4}+3V-2\Omega I_{2}=0\\ I_{2}+I_{8}=I_{4} \end{matrix}\right.$$ But...
  15. GuthrieF

    Three phase systems - current magnitude

    so far on this question i have calculated the phase voltage as 415 Volts using by multiplying the line voltage by root 3. i know how to calculate (d) but am struggling with finding the equations to satisfy
  16. S

    Calculating the current in an inductor (AC)

    I integrated the second equation given in the form (integral)di= 1/L * (integral) V *dt and I got an answer of 3.52 for the current at 0.4 seconds, anyone can let me know if this is correct?
  17. S

    Engineering Find Current Through 40 Ohms Resistor Using Thevenin

    Find the current through the 40ohms resistor using the Thevenin? I have calculated the Zth = 10 ohms. And I couldn't find the Vth.
  18. OwlsInATrenchcoat

    B Logarithmic IV graphs of diodes

    Hello there, I've been working through a task (that doesn't have an answer sheet or explanation) in which we plot I against V for three different diodes. Each has a different threshold voltage and displays the usual charcteristic curve. The final question is this: "It is suggested that the...
  19. E

    Determining the direction of current due to a change in magnetic flux

    This is supposed to be very basic, where the loop is held there is a declining magnetic field in direction (-z), therefore the current is supposed to be clockwise. However in the answers it is said the the current is anti clockwise. That doesn't make sense to me. Is there a mistake in the answers?
  20. J

    Voltage Clamp and current clamp

    If i was conducting an experiment where i wanted to determine the ion selectivity of an ion channel that has a similar structure to voltage gated Na channel but is different in ion selectivity, would i use a voltage clamp or current clamp to figure this out. Also would i use a cell line or an...
  21. E

    Why is the current the same in series circuits?

    If the current goes through the first end of the resistor will it be less than on the other end of resistor?
  22. archaic

    Current leakage between the charged plates of a capacitor

    1)$$\frac{Q}{\Delta V}=\frac{\kappa\epsilon_0A}{\ell}\Leftrightarrow\Delta V=\frac{Q\ell}{\kappa\epsilon_0A}$$$$I=\frac{A\Delta V}{\ell\rho}=\frac{Q}{\kappa\epsilon_0\rho}$$ 2) The charge is decreasing by ##\Delta Q##, so ##Q(t)=Q-\Delta Q##.$$I=\frac{\Delta Q}{\Delta t}\Leftrightarrow\Delta...
  23. cianfa72

    Relations between ##k## Currents & Voltages for Black-Box Device

    Hi, I'm aware it is an odd question. Consider a ##k##-terminal electrical device as black-box. We know from KLC and KLV that just ##k-1## currents and ##k-1## voltages are actually independent (descriptive currents and voltages). Furthermore we generally expect there exist ##k-1## relations...
  24. curiosissimo

    Using Kirchhoff's laws on this current division problem

    The main problem is the verse of the 2 currents ## i1 ## and ## i2 ## . I think they both go clockwise, so in the node A we should have ##i1+i2=20A##. Now let's apply the second law (going clockwise). So ##ΔV1-R1*i3 - R2*i4 + ΔV2=0##, where ##i3## and ##i4## are the currents which pass through...
  25. LCSphysicist

    I What are current Physicists' general positions on the Copenhagen Interpretation?

    Actually, is not a doubt as a question, in which there is wrong or right. I just want to update myself with respect to the current physicists opinion about the Copenhagen interpretation of Bohr and Heisenberg. Summarizing, there is a consensus among the majority? In another words, there is still...
  26. S

    Does the current speed matter when accelerating?

    Neglecting the air resistance, is it more difficult for a biker with a proper gearing in his bike, to increase his speed by 5 kph if his current speed is 20 kph rather then if his speed is 10 kph? It seems that the answer is yes according to the energy formula but I've asked several bikers and...
  27. E

    Does the electrostatic force create current in the circuit?

    As I understand potential difference is the reason of current. Does it mean that the electrostatic force creates current?
  28. K

    Calculating Angle Between E-Field and Current Vectors in Anisotropic Mat.

    In a certain anisotropic conductive material, the relationship between the current density ##\vec j## and the electric field ##\vec E## is given by: ##\vec j = \sigma_0\vec E + \sigma_1\vec n(\vec n\cdot\vec E)## where ##\vec n## is a constant unit vector. i) Calculate the angle between the...
  29. R

    Electrical How to wire an LED switch based on rated voltage and current

    Could someone please help. i need to connect a 12vdc motor which can draw 16 amps to a switch that has an led on it. the led is rated at 3v 20mA. The power supply i am using is rated to put out 12vdc at 30A I thought of the possibility of adding a resistor to the positive pole of the led. In...
  30. Kaushik

    Understanding the Phase Difference Between Voltage & Current in a Circuit

    Consider a circuit with a witch, capacitor and an AC voltage source. The sinusoidal AC voltage source is depicted in the following graph: We know that, ##Q = CV## ##\frac{dQ}{dt} = C \frac{dV}{dt}## ##i = C\frac{dV}{dt} \tag{1}## So from the graph, the voltage increases rapidly around ## t =...
  31. P

    Magnitude and direction of current

    I am struggling with the angles. Since the conducting wire is moving down for it to be stand still, the force should be opposing it, hence the current should be from Right to Left. I am confident of the Force direction and its value is if ## I ## is the current ##F = ILB (N)## since L and B...
  32. Nicolas01

    How to solve current circuit problems without enough information?

    https://www.physicsforums.com/attachments/267955 I think i should apply kirchkoff rules so I can find i1,i2,i3 R1,R' but I don't have enough information and I don't know how to have those informations .What is your ideas?
  33. jisbon

    Using nodal analysis to solve for current

    Hi all, Managed to get an answer out, but unfortunately, it doesn't correspond with the answer given ( –3.09 A) Would appreciate if one could check out what I did wrongly here: ## V_{1}=2\times 5=10V ## At node 2: ## \dfrac{V_{1}-V_{2}}{2}=12+\dfrac{V_{2}}{1}+\dfrac{V_{2}-V_{3}}{2}## We can...
  34. S

    Current in a current-carrying loop experiencing no torque

    Disclaimer: The solution to this question has already been posted by my instructor. I made this post to understand why my solution is wrong or if the instructor is wrong, since their explanation does not make sense to me. My reasoning: Using the fact that the magnetic torque on a...
  35. P

    Finding the current in Field winding

    The problem seems to be easy but i don't get the correct answer. a. The current in the field coils. The net resistance of Rf = 106 and Rr = 5.9 is ## Reff = \frac {(106 * 5.9)} {(106.9 + 5.9)} = 5.54 ## ## \frac { 120 - E} {5.9} = 4.82 => E = 91.562 ## ## If = \frac { 120 - 91.562} {106} =...
  36. P

    Calculating the Force and Torque on a current loop

    ##\vec \tau = \vec \mu \times \vec B -eq1## ## F = I \vec l \times \vec B -eq2 ## The forces i have indicated The magnetic moment is coming out of the page. I am only attempting only the part (a) as of now a. The force calculations are F1 = 1.4*0.22*1.5 = 0.462 +X direction F2 = 1.4*0.22*1.5...
  37. P

    Force on a current carrying conductor

    I am studying the theory of force on a current carrying conductor in a uniform magnetic field, the Force is ## F=ILB##. I am slightly confused here let us say the conductor is placed in the uniform magnetic field ##B##, then if the current is passing through the conductor then that current would...
  38. Jon Zimmer

    Relation between electric motor and battery current, peak etc.

    Good afternoon guys, I was making some researches about building my own EV and its' specifications but I have a few no direct answered questions and some of them I humbly ask the help for you guys, based on the configuration example below. Considering that I hypothetically have an electric DC...
  39. Frigus

    Why the current is maximum When Rᵉˣᵗᵉʳⁿᵃˡ=Rᴵⁿᵗᵉʳⁿᵃˡ?

    My teacher proved it like this I= ##\frac {E}{R+r}## Where R is external resistance And r is internal resistance E is emf of the cell, Now we can write it like this I= ##\frac {E}{(√R-√r)²+2√R√r}## if R=r then the denominator value will be minimum but i am unable to understand that why this is...
  40. halleff

    Diffusion current and carrier concentration equilibrium (unbiased)

    Suppose you have a non-uniformly doped piece of semiconductor (without an applied bias) such that the acceptor dopant concentration Na(x) decreases from left to right (as x increases). In this case, the equilibrium hole distribution p(x) will not be uniform since then there would be a net drift...
  41. E

    Force derived from magnetic energy of a current carrying spring

    The magnetic energy of a current carrying spring, with ##N## turns, length ##x## and cross sectional area ##A##, is $$E_m = \frac{\mu_0 N^2 I^2 A}{2x}$$The (negated) spatial derivative of this yields a quantity with dimensions of force,$$F = - \frac{dE_m}{dx} = \frac{\mu_0 N^2 I^2 A}{2x^2}$$How...
  42. tanaygupta2000

    Calculate Ip: 9V, 120V Transformer Equation

    Okay, so according to the transformer equation, VpIp = VsIs In this question, I know I have to calculate Ip. I think given, Vp = 9V, Vs = 120 V Now I don't know what to use in the value of Is. Please help! The answer is 2.32 A.
  43. tanaygupta2000

    Resultant field at the center of two semicircular current arcs

    So the magnetic field induced at the center of a current-carrying loop is given by: B = μ0 i /2r where r is the radius of the loop In the case of a semi-circular loop, this becomes B = μ0 i /4r In the question, i = 2A, r1 = 1m and r2 = 2m So, field induced at the center of first semicircular...
  44. HelloCthulhu

    What is the current during BaTiO3 voltage breakdown?

    I've been researching dielectric breakdown for a while and came across this interesting experiment: https://tore.tuhh.de/bitstream/11420/1160/1/Size_dependence_of_the_dielectric_breakdown_strength_from_nano_to_millimeter_scale_TUB_Doc_version.pdf 0.3mm of BaTiO3 was placed between two...
  45. T

    COVID Current COVID-19 widespread mutation is more infectious than the original

    Yup, it's mutated to be more contagious and has a higher viral load in patients, but does NOT change the severity of symptoms. It is a mutation that was noted back in Feb., 2020 of the original version from China, it has spread worldwide. 52 page research article in Cell ...
  46. L

    Question:What is the difference between flux and current?

    Guys,I need to calculate the flux and current of neutron by MCNP.So what's the difference between them ,and How to calculate them? Any ideas?
  47. R

    Cylindrical Conductors Carrying a Current I -- Formula (?)

    How can I' be the formula above? Is there any formula to get this same
  48. S

    Force acting on a current carrying conductor within a magnetic field

    I am new to this forum, and this is my first post. Please bear with me if my query has any inaccuracies. In the attached figure, a coil is wrapped around the central arm of a flat H-shaped thin metallic plate (such as iron). DC current flows through the coil and magnetizes the arm. At the...
Back
Top